0 Daumen
97 Aufrufe

Aufgabe: Ist diese Reihe konvergent? Für a € R und a >= 0

\( \sum\limits_{n=0}^{\infty}{\frac{1 + n^2}{n^2 +n^{3+a}})} \)

Problem/Ansatz:

Ich habe es mehr fach mit bekannten Reihen konvergenzkriterien versucht wie mit dem Quotientenkriterium aber bin zu keinem Ergebnis gekommen...

Avatar von

Das wird wohl von dem a abhängen, ob konvergent oder nicht.

Z.B. für a=-1 sind die Summanden

 \( \frac{1 + n^2}{n^2 +n^{3-1}} = \frac{1 + n^2}{n^2 +n^{2}} =  \frac{1 + n^2}{2n^2} \)

Sie bilden also keine Nullfolge, damit ist die Reihe divergent.

Habs Korrigiert

1 Antwort

0 Daumen

Hallo

1. Minor ante, im Zähler 1 weglassen , dann unten n^2 ausklammern und kürzen,   die +1 im Nenner ändert das Konvergenzverhalten nicht, dann hast du das bekannte 1/na+1

lul

Avatar von 106 k 🚀

Ein anderes Problem?

Stell deine Frage

Willkommen bei der Mathelounge! Stell deine Frage einfach und kostenlos

x
Made by a lovely community